2004 AMC 12A Problems/Problem 3

Revision as of 20:08, 17 January 2008 by Ruoke (talk | contribs)

Problem

This is an empty template page which needs to be filled. You can help us out by finding the needed content and editing it in. Thanks.

Solution

This problem needs a solution. If you have a solution for it, please help us out by adding it.

See Also

2004 AMC 12A (ProblemsAnswer KeyResources)
Preceded by
Problem 2
Followed by
Problem 4
1 2 3 4 5 6 7 8 9 10 11 12 13 14 15 16 17 18 19 20 21 22 23 24 25
All AMC 12 Problems and Solutions

The answer is (B) since x can't be 0, which leaves only 49 different values for y.